0 Daumen
688 Aufrufe

Hallo. Ich brauche Hilfe bei dieser Aufgabe:



(a) Es sei ein Wahrscheinlichkeitsraum \( (\Omega, \mathcal{A}, P) \) gegeben, sowie die Ereignisse \( A, B \) und C. Die Ereignisse \( A, B, C \) seinen stochastischunabhängig. Zeigen Sie
$$ P(A \cup B \cup C)=1-P(\bar{A}) \cdot P(\bar{B}) \cdot P(\bar{C}) $$
(b) Ein idealer Würfel werde zweimal hintereinander geworfen. Wir definieren die folgenden Ereignisse. \( A: \), die Augenzahl im ersten Wurf ist gerade",
\( B: \), die Augenzahl im zweiten Wurf ist mindestens \( 5 " \)
\( C: \), die Augensumme ist ungerade",
\( D: \), die Augenzahl im zweiten Wurf ist höchstens \( 3 \) und
\( E: \), das Produkt der Augenzahlen ist durch 5 teilbar". Untersuchen Sie jeweils, ob die angegebenen Ereignisse stochastisch unabhängig sind und ob sie paarweise stochastisch unabhängig sind. i. \( A, C, D \) ii. \( C, D, E \)

Avatar von

Was ist denn mit den Exponenten usw. bei Augenzahlen gemeint?Skärmavbild 2020-01-09 kl. 11.18.08.png

Text erkannt:

(a) Es sei ein Wahrscheinlichkeitsraum Y( MOmega, Vmathcal(A), P) V gegeben, sowie die Ereignisse \( \mathrm{V}(\mathrm{A}, \mathrm{B} \text { ) und } \mathrm{C} \text { . Die Ereignisse } \mathrm{V}(\mathrm{A}, \mathrm{B}, \mathrm{C}) \) seinen stochastischunabhängig. Zeigen Sie SS \( \mathrm{P}(\mathrm{A} \text { lcup } \mathrm{B} \text { lcup } \mathrm{C})=1-\mathrm{P}(\text { bar }\{\mathrm{A}\}) \) lodot \( \mathrm{P} \) (bar \( \{\mathrm{B}\} \) ) lodot \( \mathrm{P} \) Nbar\{Cy) SS (b) Ein idealer Würfel werde zweimal hintereinander geworfen. Wir definieren die folgenden Ereignisse. Y( A: ], die Augenzahl im ersten Wurf ist gerade", \( \mathrm{V}(\mathrm{B}: \mathrm{V}), \) die Augenzahl im zweiten Wurf ist mindestens \( \mathrm{V}\left(5^{\mathrm{n}} \mathrm{V}\right) \)
\( \mathrm{V}(\mathrm{C}: \mathrm{V}), \) die Augensumme ist ungerade" \( \mathrm{V}(\mathrm{D}: \mathrm{U}), \text { die Augenzahl im zweiten Wurf ist höchstens } \mathrm{Y}(3 \mathrm{N} 4\} \mathrm{U}) \) und V(E: U), das Produkt der Augenzahlen ist durch 5 teilbar". Untersuchen Sie jeweils, ob die angegebenen Ereignisse stochastisch unabhängig sind und ob sie paarweise stochastisch unabhängig sind. i. \( \mathrm{N}(\mathrm{A}, \mathrm{C}, \mathrm{D} \cup \text { ) ii. } \mathrm{N}(\mathrm{C}, \mathrm{D}, \mathrm{E} \vee) \)

Bitte beim Entfernen von Bildern Vorsicht walten lassen.

Da sollte nur 3 sein nicht 3^4

2 Antworten

0 Daumen

Wen A, B, C unabhängig sind, dann sind auch \(\bar{A} , \bar{B} , \bar{C}\) unabhängig und es gilt dann \(P(\bar{A}) \cdot P(\bar{B}) \cdot P(\bar{C})=P(\bar{A} \cap \bar{B} \cap \bar{C})\), und \(\bar{A} \cap \bar{B} \cap \bar{C}\) ist das Gegenereignis von  \(A \cup B \cup C\).

Avatar von 53 k 🚀
0 Daumen

Aloha :)

zu Teil a:)$$P(A\cup B\cup C)=1-P(\overline{A\cup B\cup C})=1-P(\overline A\cap\overline B\cap\overline C)$$$$\phantom{P(A\cup B\cup C)}=1-P(\overline A)\cdot P(\overline B)\cdot P(\overline C)$$Stichwort: Regeln von de Morgan für Mengen:

https://de.wikipedia.org/wiki/De_Morgansche_Gesetze


zu Teil b)$$p[A: (2,y)\;(4,y)\;(6,y)]=\frac{1}{2}$$$$p[B: (x,5)\;(x,6)]=\frac{1}{3}$$$$p[C: (x,y)\text{ mit } x+y \text{ ungerade}]=\frac{1}{2}$$$$p[D: (x,1)\;(x,2)\;(x,3)]=\frac{1}{2}$$$$P[E: (x,5)\;(5,y)]=\frac{11}{36}$$

Diese 5 Wahrscheinlichkeiten sollst du nun paarweise miteinader kombinieren. Wenn die Paare linear unabhängig sind, steht ein \(\checkmark\) am Ende, sonst ein \(\otimes\).

$$P(A\cap B)=\frac{\#\{(2,5)(2,6)(4,5)(4,6)(5,6)(6,6)\}}{36}=\frac{6}{36}=\frac{1}{6}$$$$\phantom{P(A\cap B)}=\frac{1}{2}\cdot\frac{1}{3}=P(A)\cdot P(B)\quad\checkmark$$$$P(A\cap C)=\frac{\#\{(2,1)(2,3)(2,5)(4,1)(4,3)(4,5)(6,1)(6,3)(6,5)\}}{36}=\frac{9}{36}=\frac{1}{4}$$$$\phantom{P(A\cap C)}=\frac{1}{2}\cdot\frac{1}{2}=P(A)\cdot P(C)\quad\checkmark$$$$P(A\cap D)=\frac{\#\{(2,1)(2,2)(2,3)(4,1)(4,2)(4,3)(6,1)(6,2)(6,3)\}}{36}=\frac{9}{36}=\frac{1}{4}$$$$\phantom{P(A\cap D)}=\frac{1}{2}\cdot\frac{1}{2}=P(A)\cdot P(D)\quad\checkmark$$$$P(A\cap E)=\frac{\#\{(2,5)(4,5)(6,5)\}}{36}=\frac{3}{36}=\frac{1}{12}$$$$\phantom{P(A\cap E)}\ne\frac{1}{2}\cdot\frac{11}{36}=P(A)\cdot P(E)\quad\otimes$$$$P(B\cap C)=\frac{\#\{(2,5)(4,5)(6,5)(1,6)(3,6)(5,6)\}}{36}=\frac{6}{36}=\frac{1}{6}$$$$\phantom{P(B\cap C)}=\frac{1}{3}\cdot\frac{1}{2}=P(B)\cdot P(C)\quad\checkmark$$$$P(B\cap D)=\frac{\#\{\}}{36}=\frac{0}{36}=0$$$$\phantom{P(B\cap D)}\ne\frac{1}{3}\cdot\frac{1}{2}=P(B)\cdot P(D)\quad\otimes$$$$P(B\cap E)=\frac{\#\{(1,5)(2,5)(3,5)(4,5)(5,5)(6,5)(5,6)\}}{36}=\frac{7}{36}$$$$\phantom{P(B\cap E)}\ne\frac{1}{3}\cdot\frac{11}{36}=P(B)\cdot P(E)\quad\otimes$$$$P(C\cap D)=\frac{\#\{(2,1)(4,1)(6,1)(1,2)(3,2)(5,2)(2,3)(4,3)(6,3)\}}{36}=\frac{9}{36}=\frac{1}{4}$$$$\phantom{P(C\cap D)}=\frac{1}{2}\cdot\frac{1}{2}=P(C)\cdot P(D)\quad\checkmark$$$$P(C\cap E)=\frac{\#\{(2,5)(4,5)(6,5)(5,2)(5,4)(5,6)\}}{36}=\frac{6}{36}=\frac{1}{6}$$$$\phantom{P(C\cap E)}\ne\frac{1}{2}\cdot\frac{11}{36}=P(C)\cdot P(E)\quad\otimes$$$$P(D\cap E)=\frac{\#\{(5,1)(5,2)(5,3)\}}{36}=\frac{3}{36}=\frac{1}{12}$$$$\phantom{P(C\cap D)}\ne\frac{1}{2}\cdot\frac{11}{36}=P(D)\cdot P(E)\quad\otimes$$

Avatar von 148 k 🚀

Ein anderes Problem?

Stell deine Frage

Willkommen bei der Mathelounge! Stell deine Frage einfach und kostenlos

x
Made by a lovely community